2
$\begingroup$

Assume we talk about the $n$ dimensional vector space over the reals.

It is easy to see that for any norm the unit ball is a convex symmetric set. And here is my question :

Let $A$ be a bounded , symmetric, closed, convex set. Assume that $A$ is not contained in any $k$ dimensional affine subspace for $k\leq n-1$. Show (disprove ?) that $A$ is a closed unit ball for some norm.

$\endgroup$
2
  • $\begingroup$ A is a unit ball for some norm* . $\endgroup$ Commented Jan 13, 2014 at 23:07
  • $\begingroup$ Seminorms can be related: en.wikipedia.org/wiki/… $\endgroup$
    – Berci
    Commented Jan 14, 2014 at 0:08

1 Answer 1

5
$\begingroup$

Okay, let us assume without loss of generality, our set $A$ is symmetric to $0$. Then we can just look at the closed unit ball with centre $0$. Define a function $$N\colon \mathbb{R}^n \to \mathbb{R}_+, v \mapsto \inf \left\{ r > 0 : \frac1r \cdot v \in A \right\}.$$ Now, we have to check that $N$ is a norm:

  1. For all $a \in \mathbb{R}$ and $v \in \mathbb{R}^n$: $N(av) = |a|N(v)$: \begin{align} N(av) &= \inf \left\{ r > 0 : \frac{a}r \cdot v \in A \right\} \\ &= |a| \cdot \inf \left\{ r > 0 : \frac1r \cdot v \in A \right\} &\text{Since $A$ is symmetric}\\ &= |a| \cdot N(v) \end{align}
  2. If $N(v) = 0 \Longrightarrow v = 0:$ Assume $N(v) = 0$ and $v \neq 0$. Then $\frac1r \cdot v \in A$ for all $r \in \mathbb{R}$ which contradicts our assumption that $A$ bounded. Thus, $v = 0$.
  3. For all $v, w \in \mathbb{R}^n, N(v + w) \leq N(v) + N(w):$ I think this is a bit harder to show. However, it is also the most interesting part since you need the convexity of $A$ to show subadditivity. How do we show this? Consider $v, w \in \mathbb{R}^n$ and assume that $v, w \neq 0$. By definition, we find that $v/N(v), w/N(w) \in A$. Since $A$ is convex, we know that $$\frac{v + w}{N(v)+N(w)} \in A$$ and thus $$\frac{N(v + w)}{N(v)+N(w)} \leq 1.$$ which implies that $N(v + w) \leq N(v)+N(w)$.

Last but not least we have to show that $A$ is the unit ball of $N$. Since all norms in $\mathbb{R}^n$ are continuous, $N$ is continuous (See other posts). $A \subseteq \{v \in \mathbb{R}^n: N(v) \leq 1\}$ is trivial by definition. The other inclusion $A \supseteq \{v \in \mathbb{R}^n: N(v) \leq 1\}$ follows from continuity of $N$.

$\endgroup$
3
  • $\begingroup$ well , no ... Of course i am referring to closed balls. In face,we should assume that the set A is closed (sorry!) i.e we need to prove existence of a norm in which A is the closed unit ball. $\endgroup$ Commented Jan 14, 2014 at 6:50
  • $\begingroup$ PS: I just saw Berci's comment. You might take a look at Minkowski functional en.wikipedia.org/wiki/Minkowski_functional and Minkowski Geometry by Thompson. $\endgroup$
    – Marc
    Commented Jan 14, 2014 at 9:20
  • $\begingroup$ I did the same thing (i.e any vector on the boundary would be of length 1 and we define the "norm" to be multiplicative. And indeed the first 2 things are easy (as well as it is to show the norm is well-defined) but i can't prove the triangle inequality just yet. $\endgroup$ Commented Jan 14, 2014 at 11:04

You must log in to answer this question.

Not the answer you're looking for? Browse other questions tagged .